It looks like you're new here. If you want to get involved, click one of these buttons!
In the 7Sage version of PT33 and in the Logic Games Bundle,
PT33.S4.Q14 (LG) says:
"most be true"
http://imgur.com/yPG9YaW
I paused for like three seconds figuring out what it means before realizing that this is a typo.
I think I found another one the other day, but I can't remember which PT it was from.
So please let me know if you find any typos so that @"7Sage Admin" can fix someday...
Comments
Interestingly enough, that typo was on the original PT. There are definitely a few more I've seen too. mostly from the older tests.
Hmmm....Which version are you looking at? On the page 178 of my "The Next 10 Actual, Official LSAT PrepTests" (https://www.amazon.com/Next-Actual-Official-LSAT-PrepTests/dp/0979305055 ), the Question 14 says "must be true."
Oh damn! I'm looking at Kaplan's PDF of the test.
Yup, that's a typo. Will be getting that fixed.
I found another typo! (tagging @"7Sage Admin", @"Dillon A. Wright" as well)
PT30.S1. Game 3
The fourth rule says:
http://imgur.com/C79C05W
It should be "Neither O's nor T's."
@"Dillon A. Wright"
I should probably email you, but the PT30.S2.Q13 on the e-doc of the "Flaw Or Descriptive Weakening Drill" of the "LR Drills - Preptest 30 to 39" (https://7sage.com/lesson/lr-drills-preptest-30-to-39/) does not show the latter half of the answer choice (C).
Maybe this is only happening to me, but could you maybe check this?
http://imgur.com/9TFx0qH
Weird, it shows for me. You may want to clear your cache:
http://7sage.com/clear-cache/
Weird! Thank you, I'll try!!!